The OLEX Petroleum Company has recently determined that it

This topic has expert replies
Legendary Member
Posts: 1574
Joined: Fri Jan 28, 2011 2:52 am
Thanked: 88 times
Followed by:13 members
The OLEX Petroleum Company has recently determined that it could cut its refining costs by closing its Grenville refinery and consolidating all refining at its Tasberg refinery. Closing the Grenville refinery, however, would mean the immediate loss of about 1,200 jobs in the Grenville area. Eventually the lives of more than 10,000 people would be seriously disrupted. Therefore, OLEX's decision, announced yesterday, to keep Grenville open shows that at OLEX social concerns sometimes outweigh the desire for higher profits. Which of the following, if true, most seriously undermines the argument given?

A. The Grenville refinery, although it operates at a higher cost than the Tasberg refinery, has nevertheless been moderately profitable for many years.

B. Even though OLEX could consolidate all its refining at the Tasberg plant, doing so at the Grenville plant would not be feasible.

C. The Tasberg refinery is more favorably situated than the Grenville refinery with respect to the major supply routes for raw petroleum.

D. If the Grenville refinery were ever closed and operations at the Tasberg refinery expanded, job openings at Tasberg would to the extent possible be filled with people formerly employed at Grenville.

E. Closure of the Grenville refinery would mean compliance, at enormous cost, with demanding local codes regulating the cleanup of abandoned industrial sites.

[spoiler]OA: Will be posted later. I have one doubt i.e

Assumption : OLEX didn't shut down the plant because the lives of 10,000 people would have got impacted.

Conclusion: Therefore, OLEX's decision, announced yesterday, to keep Grenville open shows that at OLEX social concerns sometimes outweigh the desire for higher profits.

Strategy : In case of assumption question I read somewhere that we should find that answer choice which when negated makes the conclusion fall apart.

Now if I use the above strategy and try to negate option A, then option A becomes

A. The Grenville refinery, although it operates at a higher cost than the Tasberg refinery, has been moderately profitable for many years. -----> isn't this option making conclusion fall apart by stating that Greenville refinery has not been shut down because though being expensive it has been profitable.

Please correct me if I am wrong.......[/spoiler]

User avatar
Legendary Member
Posts: 1325
Joined: Sun Nov 01, 2009 6:24 am
Thanked: 105 times
Followed by:14 members

by vikram4689 » Sun Jul 03, 2011 8:45 am
IMO E, Now the answer to your ques. (this is NOT an assumption ques, it is a Weakening Ques.)

First an advice, Use negation only if you are not able to solve otherwise. It is can be hazardous(yup i mean literally ;) ) if negation is not done properly.

Second, Negation is not done properly. What you is called Black-White approach i.e. you took just opposite but there are other possibilities.
A - The Grenville refinery, although it operates at a higher cost than the Tasberg refinery, has nevertheless been moderately profitable for many years.

Negation A - The Grenville refinery, although it operates at a higher cost than the Tasberg refinery, has NOT nevertheless been moderately profitable for many years.

There is possibility that A has not given profits so in that case it does NOT undermine the argument.
Premise: If you like my post
Conclusion : Press the Thanks Button ;)

Legendary Member
Posts: 1574
Joined: Fri Jan 28, 2011 2:52 am
Thanked: 88 times
Followed by:13 members

by aspirant2011 » Sun Jul 03, 2011 9:02 am
vikram4689 wrote:IMO E, Now the answer to your ques. (this is NOT an assumption ques, it is a Weakening Ques.)

First an advice, Use negation only if you are not able to solve otherwise. It is can be hazardous(yup i mean literally ;) ) if negation is not done properly.

Second, Negation is not done properly. What you is called Black-White approach i.e. you took just opposite but there are other possibilities.
A - The Grenville refinery, although it operates at a higher cost than the Tasberg refinery, has nevertheless been moderately profitable for many years.

Negation A - The Grenville refinery, although it operates at a higher cost than the Tasberg refinery, has NOT nevertheless been moderately profitable for many years.

There is possibility that A has not given profits so in that case it does NOT undermine the argument.
yup got it, u are damn good in CR's and I am damn weak in them :-)

User avatar
GMAT Instructor
Posts: 1035
Joined: Fri Dec 17, 2010 11:13 am
Location: Los Angeles, CA
Thanked: 474 times
Followed by:365 members

by VivianKerr » Sun Jul 03, 2011 4:15 pm
Here's my take on this one:

Conclusion: OLEX's decision to keep Grenville = Social concerns valued over more $$$
Evidence: Refining costs cut by closing Grenville; Grenville closing = loss of jobs/live disruption

Assumption: The disruption wouldn't lead to less profitability long-term; there isn't another reason OLEX abandoned the closing idea

Q Rephrase: What UNDERMINES the argument? (Weaken Question- we need to REVERSE the assumptions)

Prediction: Disruption = less profitability, OLEX not actually concerned w/social well-being

E perfectly matches our prediction.

You did a really nice job of trying to take the argument apart (kudos!), but as aspirant2011 mentioned, this is not an assumption question.

To review some tips on Negation Technique for Assumption questions, you might want to check this blog out: https://www.beatthegmat.com/mba/2011/06/ ... ssumptions
Vivian Kerr
GMAT Rockstar, Tutor
https://www.GMATrockstar.com
https://www.yelp.com/biz/gmat-rockstar-los-angeles

Former Kaplan and Grockit instructor, freelance GMAT content creator, now offering affordable, effective, Skype-tutoring for the GMAT at $150/hr. Contact: [email protected]

Thank you for all the "thanks" and "follows"! :-)

Newbie | Next Rank: 10 Posts
Posts: 8
Joined: Wed May 04, 2011 8:34 am
Thanked: 4 times

by er.aparna » Sun Jul 03, 2011 8:52 pm
Aspirant: The argument does not state that OLEX Petroleum Company is closing the Grenville refinery because its not profitable but because it can reduce costs (read increase profits) by moving production to Tasberg refinery. Hence, relative profitability of these two refineries and not the absolute profitability of any one refinery is in question. Your application of negation test assumes absolute profitability and hence is not correct. Choice A is therefore not correct.

In my opinion, Choices D/E are close choices. However, I prefer choice E because of 2 reasons:

1. The argument talks about disruption. Even if employeed from Grenville refinery find employment at Tasberg refinery they would still have to move which means that their lives will be disrupted.

2. Choice E states an alternate reason why OLEX Petroleum Company will face very high close up costs. Hence, this undermines that conclusion that OLEX Petroleum Company choose not to close Grenville refinery because of social concerns. Hence this is the correct choice.

Legendary Member
Posts: 2330
Joined: Fri Jan 15, 2010 5:14 am
Thanked: 56 times
Followed by:26 members

by mundasingh123 » Mon Jul 04, 2011 7:53 am
vikram4689 wrote:IMO E, Now the answer to your ques. (this is NOT an assumption ques, it is a Weakening Ques.)

First an advice, Use negation only if you are not able to solve otherwise. It is can be hazardous(yup i mean literally ;) ) if negation is not done properly.

Second, Negation is not done properly. What you is called Black-White approach i.e. you took just opposite but there are other possibilities.
A - The Grenville refinery, although it operates at a higher cost than the Tasberg refinery, has nevertheless been moderately profitable for many years.

Negation A - The Grenville refinery, although it operates at a higher cost than the Tasberg refinery, has NOT nevertheless been moderately profitable for many years.

There is possibility that A has not given profits so in that case it does NOT undermine the argument.
IMO aspirant2011 was almost right with his approach .
I think you misinterpreted what aspirant2011 said and then aspirant2011 was caught up in this maze of convoluted language
What aspirant2011 , i think wanted to say was "The reverse of A is an assumption " so if you reverse an assumption , you have a weaken .
I Seek Explanations Not Answers

Legendary Member
Posts: 1574
Joined: Fri Jan 28, 2011 2:52 am
Thanked: 88 times
Followed by:13 members

by aspirant2011 » Mon Jul 04, 2011 8:44 am
VivianKerr wrote:Here's my take on this one:

Conclusion: OLEX's decision to keep Grenville = Social concerns valued over more $$$
Evidence: Refining costs cut by closing Grenville; Grenville closing = loss of jobs/live disruption

Assumption: The disruption wouldn't lead to less profitability long-term; there isn't another reason OLEX abandoned the closing idea

Q Rephrase: What UNDERMINES the argument? (Weaken Question- we need to REVERSE the assumptions)

Prediction: Disruption = less profitability, OLEX not actually concerned w/social well-being

E perfectly matches our prediction.

You did a really nice job of trying to take the argument apart (kudos!), but as aspirant2011 mentioned, this is not an assumption question.

To review some tips on Negation Technique for Assumption questions, you might want to check this blog out: https://www.beatthegmat.com/mba/2011/06/ ... ssumptions
thanks Vivian for your post, it helped me a lot :-)

Legendary Member
Posts: 1574
Joined: Fri Jan 28, 2011 2:52 am
Thanked: 88 times
Followed by:13 members

by aspirant2011 » Mon Jul 04, 2011 8:46 am
mundasingh123 wrote:
vikram4689 wrote:IMO E, Now the answer to your ques. (this is NOT an assumption ques, it is a Weakening Ques.)

First an advice, Use negation only if you are not able to solve otherwise. It is can be hazardous(yup i mean literally ;) ) if negation is not done properly.

Second, Negation is not done properly. What you is called Black-White approach i.e. you took just opposite but there are other possibilities.
A - The Grenville refinery, although it operates at a higher cost than the Tasberg refinery, has nevertheless been moderately profitable for many years.

Negation A - The Grenville refinery, although it operates at a higher cost than the Tasberg refinery, has NOT nevertheless been moderately profitable for many years.

There is possibility that A has not given profits so in that case it does NOT undermine the argument.
IMO aspirant2011 was almost right with his approach .
I think you misinterpreted what aspirant2011 said and then aspirant2011 was caught up in this maze of convoluted language
What aspirant2011 , i think wanted to say was "The reverse of A is an assumption " so if you reverse an assumption , you have a weaken .
Hi Mundasingh,

I applied the wrong negation technique as after negation I removed nevertheless word from option A and therefore, landed on the wrong answer :-(........

Legendary Member
Posts: 2330
Joined: Fri Jan 15, 2010 5:14 am
Thanked: 56 times
Followed by:26 members

by mundasingh123 » Mon Jul 04, 2011 8:48 am
aspirant2011 wrote:
mundasingh123 wrote:
vikram4689 wrote:IMO E, Now the answer to your ques. (this is NOT an assumption ques, it is a Weakening Ques.)

First an advice, Use negation only if you are not able to solve otherwise. It is can be hazardous(yup i mean literally ;) ) if negation is not done properly.

Second, Negation is not done properly. What you is called Black-White approach i.e. you took just opposite but there are other possibilities.
A - The Grenville refinery, although it operates at a higher cost than the Tasberg refinery, has nevertheless been moderately profitable for many years.

Negation A - The Grenville refinery, although it operates at a higher cost than the Tasberg refinery, has NOT nevertheless been moderately profitable for many years.

There is possibility that A has not given profits so in that case it does NOT undermine the argument.
IMO aspirant2011 was almost right with his approach .
I think you misinterpreted what aspirant2011 said and then aspirant2011 was caught up in this maze of convoluted language
What aspirant2011 , i think wanted to say was "The reverse of A is an assumption " so if you reverse an assumption , you have a weaken .
Hi Mundasingh,

I applied the wrong negation technique as after negation I removed nevertheless word from option A and therefore, landed on the wrong answer :-(........
Nevertheless is just used to signify a reversal in the drift of the stimulus . It means what's going to follow is opposite of what preceded the word "Nevertheless" . How can someone negate nevertheless.
I Seek Explanations Not Answers

Legendary Member
Posts: 1574
Joined: Fri Jan 28, 2011 2:52 am
Thanked: 88 times
Followed by:13 members

by aspirant2011 » Mon Jul 04, 2011 8:55 am
aspirant2011 wrote:
mundasingh123 wrote:
vikram4689 wrote:IMO E, Now the answer to your ques. (this is NOT an assumption ques, it is a Weakening Ques.)

First an advice, Use negation only if you are not able to solve otherwise. It is can be hazardous(yup i mean literally ;) ) if negation is not done properly.

Second, Negation is not done properly. What you is called Black-White approach i.e. you took just opposite but there are other possibilities.
A - The Grenville refinery, although it operates at a higher cost than the Tasberg refinery, has nevertheless been moderately profitable for many years.

Negation A - The Grenville refinery, although it operates at a higher cost than the Tasberg refinery, has NOT nevertheless been moderately profitable for many years.

There is possibility that A has not given profits so in that case it does NOT undermine the argument.
IMO aspirant2011 was almost right with his approach .
I think you misinterpreted what aspirant2011 said and then aspirant2011 was caught up in this maze of convoluted language
What aspirant2011 , i think wanted to say was "The reverse of A is an assumption " so if you reverse an assumption , you have a weaken .
Hi Mundasingh,

I applied the wrong negation technique as after negation I removed nevertheless word from option A and therefore, landed on the wrong answer :-(........
nevertheless in option A sounds transitional which I also agree because initial part says though its costly to maintain but its moderately profitable........

one can negate nevertheless by introducing not in front of it, i.e it becomes not nevertheless which means its not moderately profitable either

but I am doubtful on one thing over here i.e my first part begins with although so I feel not nevertheless cant be used for negation.......

someone please clear my this doubt.....

User avatar
GMAT Instructor
Posts: 1035
Joined: Fri Dec 17, 2010 11:13 am
Location: Los Angeles, CA
Thanked: 474 times
Followed by:365 members

by VivianKerr » Tue Jul 05, 2011 6:46 am
Just to add, I would NOT attempt to use the Negation technique for Weaken Questions. ONLY for Assumption questions. Just stick to the Conclusion/Evidence/Assumption/Question Rephrase/Prediction steps and you'll be fine.
Vivian Kerr
GMAT Rockstar, Tutor
https://www.GMATrockstar.com
https://www.yelp.com/biz/gmat-rockstar-los-angeles

Former Kaplan and Grockit instructor, freelance GMAT content creator, now offering affordable, effective, Skype-tutoring for the GMAT at $150/hr. Contact: [email protected]

Thank you for all the "thanks" and "follows"! :-)

Legendary Member
Posts: 1574
Joined: Fri Jan 28, 2011 2:52 am
Thanked: 88 times
Followed by:13 members

by aspirant2011 » Tue Jul 05, 2011 7:06 am
VivianKerr wrote:Just to add, I would NOT attempt to use the Negation technique for Weaken Questions. ONLY for Assumption questions. Just stick to the Conclusion/Evidence/Assumption/Question Rephrase/Prediction steps and you'll be fine.
Thanks a lot for your post :-), please help me in concluding the approach so that it can help me-

Case 1: When we need to find assumption

Strategy: Find a option in the answer choice and negate the same, if the option most damages the conclusion then that is our assumption.......please correct me if I am wrong]

Case 2: When we need to weaken the argument

Strategy: Find an assumption and reverse the same i.e basically rephrase. Find an answer choice which talks completely opposite of the rephrased assumption and that particular option would be the answer.....please correct me if I am wrong

Anticipating your response on the above two queries.

User avatar
GMAT Instructor
Posts: 1035
Joined: Fri Dec 17, 2010 11:13 am
Location: Los Angeles, CA
Thanked: 474 times
Followed by:365 members

by VivianKerr » Tue Jul 05, 2011 7:20 am
Case #1 is correct. If we need to find the assumption, we reverse the answer choices and look for one that hurts the argument the most. Again, make sure you're methodical and take the argument apart first so you understand the conclusion before hitting the answer-choices (https://www.beatthegmat.com/mba/2011/06/ ... ssumptions).

Case #2. There is more than one way to weaken an argument, but typically the best way is to identify the assumption that is necessary to the conclusion, and then look for its reversal in the answer choices. So, yes, your thinking is correct here. Just makes sure you WRITE DOWN your reversed assumption as your PREDICTION. It will help you eliminate.

For example:

Conclusion: All trees in Forest A should be cut down right now.
Evidence: A virus has been found in some of the trees that destroys lumber.
Assumption: The virus will quickly spread to all the trees, making all the lumber useless.

Question: What would WEAKEN the argument?
Prediction: If the virus would NOT quickly spread (then there would be no need to cut the trees down "now").
Vivian Kerr
GMAT Rockstar, Tutor
https://www.GMATrockstar.com
https://www.yelp.com/biz/gmat-rockstar-los-angeles

Former Kaplan and Grockit instructor, freelance GMAT content creator, now offering affordable, effective, Skype-tutoring for the GMAT at $150/hr. Contact: [email protected]

Thank you for all the "thanks" and "follows"! :-)

Legendary Member
Posts: 1574
Joined: Fri Jan 28, 2011 2:52 am
Thanked: 88 times
Followed by:13 members

by aspirant2011 » Tue Jul 05, 2011 9:38 am
VivianKerr wrote:

For example:

Conclusion: All trees in Forest A should be cut down right now.
Evidence: A virus has been found in some of the trees that destroys lumber.
Assumption: The virus will quickly spread to all the trees, making all the lumber useless.

Question: What would WEAKEN the argument?
Prediction: If the virus would NOT quickly spread (then there would be no need to cut the trees down "now").
Hi Vivian,

Thanks a lot for your good explanation........I couldn't understand one thing i.e in above example the part in bracket i.e (then there would be no need to cut the trees down "now"). has been used in which context.......is this a conclusion which we get after reversing the assumption???? also clear one thing i.e would answer choice hurt the reversed assumption i.e prediction or would it support the reversed assumption i.e prediction..........

User avatar
GMAT Instructor
Posts: 1035
Joined: Fri Dec 17, 2010 11:13 am
Location: Los Angeles, CA
Thanked: 474 times
Followed by:365 members

by VivianKerr » Wed Jul 06, 2011 7:21 am
@aspirant2011 The Prediction should match the correct answer choice as closely as possible. That is why it's important to write one down whenever possible. You write down a Prediction because it clarifies for you what you should look for in the answer choices.

Here, I got the Prediction by reversing the Assumption. The assumption was that the "virus would quickly spread" so that needs to be true in order for the Conclusion to be true ("trees should be cut down now"). Since it's a weaken question, we are looking for the opposite of that assumption. The opposite is that the "virus will NOT quickly spread". The part in the parentheses is just further elaboration which ties this assumption back to the conclusion.
Vivian Kerr
GMAT Rockstar, Tutor
https://www.GMATrockstar.com
https://www.yelp.com/biz/gmat-rockstar-los-angeles

Former Kaplan and Grockit instructor, freelance GMAT content creator, now offering affordable, effective, Skype-tutoring for the GMAT at $150/hr. Contact: [email protected]

Thank you for all the "thanks" and "follows"! :-)